In an experiment, some volunteers were assigned to take aerobics classes and others to take weight-training classes. ...

Stephen on November 4, 2019

Please explain

How is (E) the answer?

Reply
Create a free account to read and take part in forum discussions.

Already have an account? log in

Skylar on November 4, 2019

@StephenM Happy to help!

This is a Strengthen with Necessary Premise question. So, we are looking for the answer choice that both strengthens the argument and, if negated, would make the conclusion fall apart.

The negation of (E) is: On average, the volunteers assigned to the aerobics classes DID NOT get a greater amount of aerobic exercise overall during the experiment, including exercise outside the classes, than did the volunteers assigned to the weight-training classes.

This would make the conclusion that the passage draws illogical, because the passage takes the lower stress symptoms of members in the aerobic class to mean that aerobic exercise helps the body handle psychological stress. However, if the members of the aerobic class got less overall aerobic exercise than the weight-training group- who had higher stress symptoms- did, the conclusion would not follow. In order to work, the passage must assume that the aerobic class is the group that accurately represents the effect of aerobics. Therefore, the argument requires (E) as an assumption.

(A) is incorrect because of the word "fully."
(B) is incorrect because outside weight-lifting is irrelevant as the passage's focus is on aerobics.
(C) is incorrect because it does not matter if the aerobics group is getting MORE exercise with the class than before, as long as they're in the class.
(D) is incorrect because it does not necessarily strengthen the passage's conclusion. One could argue that it weakens it by providing an alternate explanation for the lowered stress.

Does this make sense? Please let us know if you have additional questions!